Vous êtes sur la page 1sur 9

nguyen (kdn444) Review 1 guntel (55200) 1

This print-out should have 17 questions.


Multiple-choice questions may continue on
the next column or page nd all choices
before answering.
001 10.0 points
A car starts out heading north from Austin
on IH 35. Its velocity t hours after leaving
Austin is given (in miles per hour) by
v(t) = 5 3t
3
10
t
2
What will be the position of the car after 10
hours of driving?
1. 300 miles south of Austin
2. 225 miles south of Austin
3. 200 miles north of Austin
4. 225 miles north of Austin
5. 300 miles north of Austin
6. 275 miles north of Austin
7. 275 miles south of Austin
8. 200 miles south of Austin correct
Explanation:
Since the car leaves Austin at time t = 0,
its position t hours later is the anti-derivative
s(t) =
_
(5 3t
3
10
t
2
) dt, s(0) = 0
of v(t). But
_
(5 3t
3
10
t
2
) dt
= 5t
3
2
t
2

1
10
t
3
+ C .
On the other hand,
s(0) = 0 = C = 0.
Thus
s(t) = 5t
3
2
t
2

1
10
t
3
.
At t = 10, therefore, s(10) = 200 , the
negative sign indicating that the car is
200 miles south of Austin
after 10 hours of driving.
keywords: velocity, position, denite integral,
quadratic function
002 10.0 points
The shaded region in
x
y
is enclosed by the graphs of
f(y) = y
2
, g(y) = 3 2y .
Find the area of this shaded region.
1. area =
32
3
sq. units correct
2. area = 10 sq. units
3. area =
31
3
sq. units
4. area = 11 sq. units
5. area =
34
3
sq. units
Explanation:
The area of the enclosed region is given by
the integral
I =
_
b
a
{g(y) f(y)} dy
nguyen (kdn444) Review 1 guntel (55200) 2
where a, b are the y-coordinates of the points
of intersection of the two graphs. Now a, b
are the solutions of the equation f(y) = g(y),
i.e.,
f(y) g(y) = y
2
+ 2y 3
= (y + 3)(y 1) = 0 .
Thus
I =
_
1
3
_
3 2y y
2
_
dy
=
_
3y y
2

1
3
y
3
_
1
3
.
Consequently,
area =
32
3
sq. units .
003 10.0 points
Find the volume, V , of the solid obtained
by rotating the region bounded by
y =
3
x
, x = 2, x = 6, y = 0
about the x-axis.
1. V = 3
2. V =
3
2

3. V =
3
4
4. V =
3
4

5. V = 3 correct
6. V =
3
2
Explanation:
The volume of the solid of revolution ob-
tained by rotating the graph of y = f(x) on
[a, b] about the x-axis is given by
volume =
_
b
a
f(x)
2
dx .
When
f(x) =
3
x
, a = 2, b = 6 ,
therefore,
V =
_
6
2
9
x
2
dx .
Consequently,
V =
_

9
x
_
6
2
= 3 .
004 10.0 points
Evaluate the denite integral
I =
_
6
0
tan
1
x
6
dx .
1. I = 3 ( 2)
2. I =
3
2
( 2 ln2) correct
3. I = 3 ( 1)
4. I = 3
5. I = 6
6. I =
3
2
( + 2 ln2)
Explanation:
Let x = 6 w; then dx = 6 dw while
x = 0 = w = 0,
x = 6 = w = 1 .
In this case,
I = 6
_
1
0
tan
1
wdw ,
nguyen (kdn444) Review 1 guntel (55200) 3
so after integration by parts,
I = 6
_
w tan
1
w
_
1
0
6
_
1
0
w
1 + w
2
dw
= 6
_
w tan
1
w
1
2
ln
_
1 + w
2
_
_
1
0
.
Consequently,
I = 6
_

4

1
2
ln 2
_
=
3
2
( 2 ln2) .
keywords: integration by parts, inverse trig
function
005 10.0 points
Evaluate the denite integral
I =
_
/2
0
(3 cos x + 4 sinx) dx .
1. I = 4
2. I = 5
3. I = 3
4. I = 6
5. I = 7 correct
Explanation:
By the Fundamental Theorem of Calculus,
I =
_
F(x)
_
/2
0
= F
_

2
_
F(0)
for any anti-derivative F of
f(x) = 3 cos x + 4 sin x .
Taking
F(x) = 3 sinx 4 cos x
and using the fact that
cos 0 = sin

2
= 1,
sin 0 = cos

2
= 0 ,
we thus see that
I = 7 .
006 10.0 points
Evaluate the integral
_
e
4x
+ e
4x
(e
4x
e
4x
)
6
dx.
1.
1
20
_
e
4x
e
4x
_
5
+ C
2.
1
24
_
e
4x
e
4x
_
5
+ C
3.
1
5
_
e
4x
e
4x
_
5
+ C
4.
1
20
_
e
4x
e
4x
_
5
+ C correct
5.
1
20
_
e
4x
+ e
4x
_
5
+ C
Explanation:
Set u = e
4x
e
4x
. Then
du = 4(e
4x
+ e
4x
) dx,
so
_
e
4x
+ e
4x
(e
4x
e
4x
)
6
dx
=
1
4
_
1
u
6
du =
1
20 u
5
+ C.
Thus
_
e
4x
+ e
4x
(e
4x
e
4x
)
6
dx
=
1
20
(e
4x
e
4x
)
5
+ C
with C an arbitrary constant.
007 10.0 points
nguyen (kdn444) Review 1 guntel (55200) 4
Evaluate the integral
I =
_
2

2
2
x
2

x
2
1
dx .
1. I =

3 +

2
2. I = 2(

3 +

2 )
3. I =
1
2
(

2 )
4. I =

2 correct
5. I = 2(

2 )
6. I =
1
2
(

3 +

2 )
Explanation:
Set x = sec u. Then
dx = sec utan udu, x
2
1 = tan
2
u,
while
x =

2 = u =

4
,
x = 2 = u =

3
.
In this case,
I = 2
_
/3
/4
sec u tan u
sec
2
u tan u
du
=
_
/3
/4
2 cos udu = 2
_
sin u
_
/3
/4
.
Consequently,
I =

2 .
008 10.0 points
Evaluate the integral
I =
_
/2
0
(2 cos
2
x + sin
2
x) dx
1. I =
3
4
correct
2. I = 3
3. I =
3
2
4. I =
3
4
5. I = 3
6. I =
3
2

Explanation:
Since
cos
2
x =
1
2
(1+cos 2x), sin
2
x =
1
2
(1cos 2x) ,
we see that
2 cos
2
x + sin
2
x =
1
2
_
3 + cos 2x
_
.
Thus
I =
1
2
_

0
_
3 + cos 2x
_
dx
=
1
2
_
3x +
1
2
sin 2x
_
/2
0
.
Consequently,
I =
3
4
.
009 10.0 points
Evaluate the integral
I =
_
/2
0
(x
2
+ 7) sinx dx .
1. I =

2
7
2. I = + 5 correct
3. I =

2
+ 7
nguyen (kdn444) Review 1 guntel (55200) 5
4. I = + 7
5. I = 5
6. I =

2
+ 5
Explanation:
After integration by parts,
I =
_
(x
2
+ 7) cos x
_
/2
0
+
_
/2
0
cos x
_
d
dx
(x
2
+ 7)
_
dx
= 7 + 2
_
/2
0
x cos x dx .
To evaluate this last integral we need to inte-
grate by parts once again. For then
_
/2
0
x cos x dx =
_
x sinx
_
/2
0

_
/2
0
sin x dx =

2
+
_
cos x
_
/2
0
.
Consequently,
I = + 5 .
010 10.0 points
Determine the integral
I =
_
1
0
t
3
1 + t
8
dt .
1. I =
1
8

2. I =
1
16
correct
3. I =
1
4
4. I =
1
12

5. I =
1
4

6. I =
1
16
Explanation:
Since
_
1
1 + x
2
dx = tan
1
x + C ,
we need to reduce I to this form by changing
t. Indeed, set x = t
4
. Then
dx = 4t
3
dt ,
while
t = 0 = x = 0
t = 1 = x = 1 .
In this case
I =
1
4
_
1
0
1
1 + x
2
dx =
_
1
4
tan
1
x
_
1
0
.
Consequently,
I =
1
16
.
keywords:
011 10.0 points
Find the value of the denite integral
I =
_
/4
0
_
7 sec
4
x 8 sec
2
x
_
tanx dx .
1. I =
7
4
2. I =
5
4
correct
3. I =
13
4
4. I =
11
4
5. I =
9
4
nguyen (kdn444) Review 1 guntel (55200) 6
Explanation:
Since
d
dx
sec x = sec x tanx,
the substitution u = sec x is suggested. For
then
du = sec x tanx dx,
while
x = 0 = u = 1,
x =

4
= u =

2 .
In this case
I =
_
/4
0
_
7 sec
3
x 8 sec x
_
sec x tanx dx
=
_

2
1
(7u
3
8u) du
=
_
7
4
u
4
4u
2
_

2
1
.
Consequently,
I =
5
4
.
012 10.0 points
Evaluate the integral
I =
_
/3
0
sin x
7 6 cos x
dx .
1. I =
1
6
ln 4 correct
2. I =
1
12
ln 4
3. I = ln 4
4. I =
1
6
ln 8
5. I = ln 8
6. I =
1
12
ln 8
Explanation:
Since the integrand is of the form
sin xf(cos x) where
f(cos x) =
1
7 6 cos x
,
the substitution
u = 7 6 cos x
is suggested (or x = cos u which would do
just as well). For then du = 6 sinx dx, while
x = 0 = u = 1,
x =

3
= u = 4.
In this case,
I =
1
6
_
4
1
1
u
du =
1
6
_
ln u
_
4
1
.
Consequently,
I =
1
6
ln4 .
013 10.0 points
Evaluate the denite integral
I =
_
1
0
2x
2
3x + 4
x
2
x 2
dx .
1. I = 2 + 4 ln2
2. I = 2 5 ln2 correct
3. I = 3 + 4 ln2
4. I = 2 4 ln2
5. I = 3 5 ln2
6. I = 3 + 5 ln2
Explanation:
nguyen (kdn444) Review 1 guntel (55200) 7
By division,
2x
2
3x + 4
x
2
x 2
=
2(x
2
x 2) x + 8
x
2
x 2
= 2
x 8
x
2
x 2
.
But by partial fractions,
x 8
x
2
x 2
=
3
x + 1

2
x 2
.
Thus
I =
_
1
0
_
2
3
x + 1
+
2
x 2
_
dx .
Now
_
1
0
3
x + 1
dx =
_
3 ln|x + 1|
_
1
0
,
while
_
1
0
2
x 2
dx =
_
2 ln|x 2|
_
1
0
.
Consequently,
I =
_
2x ln

(x + 1)
3
(x 2)
2

_
1
0
= 2 5 ln2 .
014 10.0 points
Evaluate the integral
I =
_
/4
0
4 sec
4
x dx .
1. I = 5
2. I =
17
3
3. I =
16
3
correct
4. I =
14
3
5. I =
13
3
Explanation:
Since
(tanx)

= sec
2
x , sec
2
x = 1 + tan
2
x ,
use of the substitution u = tan x is suggested.
For then
du = sec
2
x dx ,
while
x = 0 = u = 0 ,
x =

4
= u = 1 .
Thus
I = 4
_
/4
0
(tan
2
x + 1) sec
2
x dx
= 4
_
1
0
(u
2
+ 1) du = 4
_
1
3
u
3
+ u
_
1
0
.
Consequently,
I =
16
3
.
015 10.0 points
Evaluate the integral
I =
_
1/2
1
2

8 2x x
2
dx .
1. I =
1
3
correct
2. I =
1
2

3
3. I =
2
3

3
4. I =
1
2

5. I =
2
3

6. I =
1
3

3
nguyen (kdn444) Review 1 guntel (55200) 8
Explanation:
By completing the square we see that
8 2 x x
2
= 9 (x + 1)
2
,
so
I =
_
1/2
1
2
_
9 (x + 1)
2
, dx .
Now set x + 1 = 3 sin u. Then
dx = 3 cos udu,
while
x = 1 = u = 0,
x =
1
2
= u =

6
.
Thus
I = 2
_
/6
0
3 cos u
3 cos u
du =
_
2u
_
/6
0
.
Consequently,
I =
1
3
.
016 10.0 points
Determine F

(x) when
F(x) =
_

x
3
2 sint
t
dt .
1. F

(x) =
2 cos(

x)

x
2. F

(x) =
2 sinx

x
3. F

(x) =
sin x
x
4. F

(x) =
2 cos x
x
5. F

(x) =
cos(

x)
x
6. F

(x) =
2 cos x

x
7. F

(x) =
sin(

x)
x
correct
8. F

(x) =
sin(

x)

x
Explanation:
By the Fundamental Theorem of Calculus
and the Chain Rule,
d
dx
_
_
g(x)
a
f(t) dt
_
= f(g(x))g

(x) .
When
F(x) =
_

x
3
2 sin t
t
dt ,
therefore,
F

(x) =
2 sin(

x)

x
_
d
dx

x
_
.
Consequently,
F

(x) =
sin(

x)
x
,
since
d
dx

x =
1
2

x
.
017 10.0 points
The graph of f is shown in the gure
-3
-2
-1
0
1
2
3
4
5
6
2 4 6 8 10
2
4
6
2
If
g(x) =
_
x
3
f(t) dt,
for what value of x does g(x) have a maxi-
mum?
nguyen (kdn444) Review 1 guntel (55200) 9
1. x = 8
2. not enough information given
3. x = 9
4. x = 4.5
5. x = 7 correct
6. x = 3
Explanation:
By the Fundamental theorem of calculus, if
g(x) =
_
x
3
f(t) dt,
then g

(x) = f(x). Thus the critical points


of g occur at the zeros of f, i.e., at the x-
intercepts of the graph of f. To determine
which of these gives a local maximum of g we
use the sign chart
g

+
3 7 9
for g

. This shows that the maximum value of


g occurs
at x = 7
since the sign of g

changes from positive to


negative at x = 7.

Vous aimerez peut-être aussi